Difference between revisions of "1955 AHSME Problems/Problem 25"

(Created page with "== Problem 25== One of the factors of <math>x^4+2x^2+9</math> is: <math> \textbf{(A)}\ x^2+3\qquad\textbf{(B)}\ x+1\qquad\textbf{(C)}\ x^2-3\qquad\textbf{(D)}\ x^2-2x-3\qqu...")
 
(2 intermediate revisions by one other user not shown)
Line 11: Line 11:
 
In addition, <math>x + 1</math> also fails the test, and that takes down <math>x^2 - 2x - 3</math>, which can be expressed as <math>(x + 1)(x - 3)</math>.
 
In addition, <math>x + 1</math> also fails the test, and that takes down <math>x^2 - 2x - 3</math>, which can be expressed as <math>(x + 1)(x - 3)</math>.
  
So the answer must be <math>\boxed{\textbf{(E)}}</math>.
+
 
 +
==Solution 2 (direct factorization)==
 +
Notice the leading and constant terms are begging us to create a binomial. So
 +
<cmath>
 +
x^4 + 2x^2 + 9 = (x^4 + 6x^2 + 9) - 4x^2 = (x^2 + 3)^2 - (2x)^2 = (x^2 + 2x + 3)(x^2 - 2x + 3),
 +
</cmath>
 +
where both quadratics are irreducible (over the field of real numbers).
 +
Hence none of the given options is a factor. So the answer is <math>\boxed{(\textbf{C})}</math>
 +
 
 +
~VensL
 +
==See Also==
 +
 
 +
{{AHSME 50p box|year=1955|num-b=24|num-a=26}}
 +
 
 +
{{MAA Notice}}

Revision as of 11:31, 10 December 2021

Problem 25

One of the factors of $x^4+2x^2+9$ is:

$\textbf{(A)}\ x^2+3\qquad\textbf{(B)}\ x+1\qquad\textbf{(C)}\ x^2-3\qquad\textbf{(D)}\ x^2-2x-3\qquad\textbf{(E)}\ \text{none of these}$

Solution

We can test each of the answer choices by using polynomial division.

$x^2 + 3$ leaves behind a remainder, and so does $x^2 - 3$.

In addition, $x + 1$ also fails the test, and that takes down $x^2 - 2x - 3$, which can be expressed as $(x + 1)(x - 3)$.


Solution 2 (direct factorization)

Notice the leading and constant terms are begging us to create a binomial. So \[x^4 + 2x^2 + 9 = (x^4 + 6x^2 + 9) - 4x^2 = (x^2 + 3)^2 - (2x)^2 = (x^2 + 2x + 3)(x^2 - 2x + 3),\] where both quadratics are irreducible (over the field of real numbers). Hence none of the given options is a factor. So the answer is $\boxed{(\textbf{C})}$

~VensL

See Also

1955 AHSC (ProblemsAnswer KeyResources)
Preceded by
Problem 24
Followed by
Problem 26
1 2 3 4 5 6 7 8 9 10 11 12 13 14 15 16 17 18 19 20 21 22 23 24 25 26 27 28 29 30 31 32 33 34 35 36 37 38 39 40 41 42 43 44 45 46 47 48 49 50
All AHSME Problems and Solutions


The problems on this page are copyrighted by the Mathematical Association of America's American Mathematics Competitions. AMC logo.png